Trang chủ Lớp 11 SBT Toán 11 Nâng cao Câu 3.3 trang 86 sách bài tập Đại số và Giải tích...

Câu 3.3 trang 86 sách bài tập Đại số và Giải tích 11 Nâng cao: Chứng minh rằng với mọi số nguyên dương n, ta luôn có các bất đẳng thức sau:...

Câu 3.3 trang 86 sách bài tập Đại số và Giải tích 11 Nâng cao.                 \({1 \over {k + 1}} + {1 \over {k + 2}} + … + {1 \over {3k + 1}} + {1 \over {3k + 2}}. Bài 1: Phương pháp quy nạp toán học

Advertisements (Quảng cáo)

Chứng minh rằng với mọi số nguyên dương n, ta luôn có các bất đẳng thức sau:

a) \({1 \over {n + 1}} + {1 \over {n + 2}} + … + {1 \over {3n + 1}} > 1\)

b) \({1 \over 2}.{3 \over 4}.{5 \over 6}…{{2n + 1} \over {2n + 2}} < {1 \over {\sqrt {3n + 4} }}\)

a) Ta sẽ chứng minh

\({1 \over {n + 1}} + {1 \over {n + 2}} + … + {1 \over {3n + 1}} > 1\)                                (1)

Với mọi \(n \in N^*,\) bằng phương pháp quy nạp.

Với \(n = 1,\) ta có

              \({1 \over 2} + {1 \over 3} + {1 \over 4} = {{13} \over {12}} > 1.\)

Như vậy, (1) đúng khi \(n = 1.\)

Giả sử đã có (1) đúng khi \(n = k,k \in {N^ * }\), tức là

              \({1 \over {k + 1}} + {1 \over {k + 2}} + … + {1 \over {3k + 1}} > 1,\)

Ta chứng minh (1) cũng đúng khi \(n = k + 1,\) nghĩa là ta sẽ chứng minh

                \({1 \over {k + 1}} + {1 \over {k + 2}} + … + {1 \over {3k + 1}} + {1 \over {3k + 2}} + {1 \over {3k + 3}} + {1 \over {3k + 4}} > 1\)

Thật vậy, ta có

     \({1 \over {k + 1}} + {1 \over {k + 2}} + … + {1 \over {3k + 1}} + {1 \over {3k + 2}} + {1 \over {3k + 3}} + {1 \over {3k + 4}}\)

\( = {1 \over {k + 1}} + {1 \over {k + 2}} + … + {1 \over {3k + 1}} + {1 \over {3k + 2}} + {1 \over {3k + 3}} + {1 \over {3k + 4}}\)

\(- {1 \over {k + 1}}\)

\( = {1 \over {k + 1}} + {1 \over {k + 2}} + … + {1 \over {3k + 1}} + {2 \over {3(k + 1)(3k + 2)(3k + 4)}}\)

Advertisements (Quảng cáo)

\( > {1 \over {k + 1}} + {1 \over {k + 2}} + … + {1 \over {3k + 1}} > 1\)  (theo giả thiết quy nạp).

Từ các chứng trên suy ra (1) đúng với mọi \(n \in N^*\)

b) Ta sẽ chứng minh

            \({1 \over 2}.{3 \over 4}.{5 \over 6}…{{2n + 1} \over {2n + 2}} < {1 \over {\sqrt {3n + 4} }}\)

     Với mọi \(n \in N^*,\) bằng phương pháp quy nạp.

Với \(n = 1,\) ta có

\({1 \over 2}.{3 \over 4} = {3 \over 8} < {1 \over {\sqrt {3.1 + 4} }}\) (  vì \(9.7 = 63 < 64 = {8^2}\) ).

Như vậy, (2) đúng khi \(n = 1.\)

Giả sử có (2) đúng khi \(n = k,k \in {N^ * }\). Khi đó, ta có

             \({1 \over 2}.{3 \over 4}.{5 \over 6}…{{2k + 1} \over {2k + 2}}.{{2k + 3} \over {2k + 4}} < {1 \over {\sqrt {3n + 4} }}.{{2k + 3} \over {2k + 4}}\,\,\,\,\,\,\,\,\,\,\,\,\,\,\,\,\,\,\,\,\,\,(3)\)   

Lại có : \({(2k + 3)^2}.(3k + 7) < {(2k + 3)^2}.(3k + 7) + k + 1\)

\(= (3k + 4){(2k + 4)^2}.\)

Do đó :      \({1 \over {\sqrt {3n + 4} }}.{{2k + 3} \over {2k + 4}} < {1 \over {\sqrt {3n + 7} }}.\,\,\,\,\,\,\,\,\,\,\,\,\,\,\,\,\,\,\,\,\,\,\,\,\,\,\,\,\,\,\,\,\,\,\,\,\,\,\,(4)\)

Từ (3) và (4) suy ra

                                \({1 \over 2}.{3 \over 4}.{5 \over 6}…{{2k + 1} \over {2k + 2}}.{{2k + 3} \over {2k + 4}} < {1 \over {\sqrt {3k + 7} }},\)

Nghĩa là ta cũng có (2) đúng khi \(n = k + 1.\)

Từ các chứng minh trên suy ra (2) đúng với mọi \(n \in N^*\).